You are on page 1of 10
Ob mau ue TuLustration 2.12 [AMses is the angle between the following pair of vectors ? Wai+j+k and B=-2)-27-26. be A. Solution. A-B = ABcos@ or cos ao .-@) But, 4.B=()+j+h)-(-2i-27-2h) A-B=-2-2-2=-6 Again 4 =| 4|= J(1)* +(1)? +0)" pei SE te =23 Now, e080 = ea => 6=180° vectos So, let us first determine Ax B yw AxB =i -S]+10k) x (61 +5] 42% Now 4x8 5j-+ 108) x (61 +5} +2k) ijk =|3 -5 10| Jo 5 2] 0-50) + j(60-6)+4(15 +30) 601 +54j-+4sk y+ (54) + (45) (RSa1 Magnitude: [4% vBsa1 So, required unit vector: fe (insane suns tatsets i +27—4k are two vectors. 1, A=21+4j7+4k and B Find the angle between them. +3)—h and 41-6) Ab srepantie 2, Iftwo vectors to each other, then find the value of 2. 3, InQ. 2, if vectors are perpendicular to each other then find the value of 2. i+3j+4k, then find the f437-h and B 4k projection of A on B 5. A body, acted upon by a force of 50 N, is displaced through a distance 10 m in a direction making an angle of 60° with the force. Find the work done by the force, es from position 3i+2}-6k to uniform force of 47+ j-+3kN. then find the work done. 6. A particle mov 147-+137+9% due toa If the displacement is in meters, 1, fortwo vectors A and B ,sum (4+) isperpendicnar tothe difference (A-B). Find the ratio of theirmagaitude. So.veo EXAMPLES. it of three vectors two are . Given that A+ B+C=0 out equal in magnitude and the magnitude of third vector ff the two having equal is V2 times that of either of g magnitude. Then the angles between vec‘ors are given by (a) 30°, 60°, 90° (b) 45°, 45°, 90° (o) 45°, 60°, 90° (a) 90°, 135% 135° 2u ceo ea ee law, three vectors having summation: Tena form a stosed polygon (Triangle) Since the two eae raving same magnitude and 4 That is, the ti e tiaale shouldbe ri angled triangle ° Pe seh gy : Angle between and 2, Angle between B and C, B 4 t “Angle between d and C7 2. A vector @ is tumed without a che it uimed without a change io its fength through a small angle d@ The value of [33] and Aa are respectively (a) 0,ad0 (c) 0,0 Sok (8) From the figure [Wal=4 and [i= (b) ad6,0 (@) None of these Also from triangle rule OB - OA: So aa ‘Aa means change in magnitude of vector, ie., |OB|~|O4l = a-a=0 So Aa=0 3. Given that 44+8=€ and that C is perpendicular to ete wht i the angle Between A, Further if [A] Aand B? z Ben (@ & min (o) & saan (@ & radian (@) radian Sol.() A=C 7 8 ‘Angle between Aand B is 130-0 =180°-45°=135°= “End pair of the following forces will never give 4. Which resultant force of 2 N? (a) 2Nand2N (&) 1 Nand 1N (@) 1Nand 4N (© 1Nand3N fo vectors 4 and B are given then range of their Sol. (d) If we eoultant can be written as (4~B)< RS(4+B). -B je. Ram = At Band Rain 212 - IfB =| and A =4 then their resultant will lies in between 3.N and 5 N. Itcan never be 2 N. Ifthese three vectors are represented by three sides of triangle then they form equilateral triangle, 5. If the sum of two unit vectors is a unit vector, then magnitude of difference is @ 2 5 © Wi @% Sol (6) Let fi, and fy are the two unit vectors, then the sum 2npcos 8 +142 c08 Since itis given that n, is also a unit vector, therefore 1=1+1+2c0s 6 => cos = 3 a O= 120° Now the difference vector is ay=hnhy or 3=n{ +n} —2nn, cos O= 1 +1 —2 cos(120°) fo We2-2012)=24183 3 ne 6. The sum of the magnitudes of two forces acting at point is 18 and the magnitude of their resultant is 12. If the resultant is at 90° with the force of smaller magnitude, what are the magnitudes of forces? (@) 12,5 (b) 14,4 (©) 5,13 @ 10,8 Sol. (c) Let P be the smaller force and Q be the greater force, then according to the problem P+O=18 R=\P? +0? +2PO cos 6 =12 wii) Qsin@ = tan 90=00 {n= Ocosd c P+Qcos 8=0 “ By solving (), (ii) and (ii) we will get P= 5, and Q= 13 7. A person moves 30 m north and then 20 m towards east and finally 30y2 m in south-west direction. The displacement of the person from the origin will be (2) 10 malong north (b) 10 m long south (©) 10matong west (d) Zero +30}, AB =201+0) ‘Sok (c) From figure, i= Physics ox BC =—30Y2 cos 45°F ~ 302 sin 45° = 36) 39} Net displacement, OC = 04+ 05+ BC=-10} +0} JOC\=10m_ 8. If a vector 2i+37+8k is perpendicular to the vector 4j—4i +ak. Then the value of ais (@® -1 wt 2 1 oe 1 © a @t Sol. (c) Given vectors can be rewritten as 7=21+3)+8% and Ba-ais4j+ad Dot product of these vectors should be equal to zero because they are perpendicular. 8+12+80=0 = Ba=-4 = a= v2 9. The vector sum of two forces is perpendicular to their vector differences. In that case, the forces (@) are equal to each other in magnitude () are not equal to each other in magnitude (c) cannot be predicted (@) are equal to each other Sol (a) If two vectors are perpendicular then their dot product rust be equal to zero. According to problem = .. A=Bie.,two vectors are equal to each other in magnitude. 10. If fortwo vector A and B, sum (4 + B) is perpendicular to the difference (4-8), The ratio of their magnitude is @ 1 (b) 2 @3 (d) None of these Sol. (a) (+B) is perpendicular to (i~ By. Thus (+B) G-By=0 or A+ BAA B- B= Because of commutative property of dot product AB 00rd B Thus the ratio of magnitudes 4/ ecto _ saat dc B=, ten wc of he flowing em ue enti @ E14 (o) C1LB @ €14+8) (a) CL (AxB) sty of vector product, we notice that C dicular to the plane formed by vector 4 and B dicular to both A and B and (A+B) vector 1e formed by vector A and B. Thus ‘sok (i) From the Prope must be perpen ‘thos @ is perpen also must lie in the must be perpendict which cannot be plan jar to (+B) but the cross product (4 xB) perpendicular to itself. Thus gives a vector é the last statement is WTONE ID. A vecior 4 points vertically upward and B points AxB is ‘The vector product (b) Along west (@ Vertically downward axis towards north. (a) Zero (o) Along east (6) Direction of vector 3 Asak Direction of vector B is tows Bab Ais alon: ‘Sol. ards north ak xj =a0-)) Now 4x . The directi Vector Addition Z ‘ are 3,4 and 5 gle between 4 vectors 4B and é ‘The rhagnitudes of ¢ A+ BHC, the ant units respectively: 1 and B is @ 2 (b) cos'0.8) 7 a ch a = (©) tan (2) OF 2. Five equal forces of 10 N each are applied at one point ‘and all are lying in one plane: ithe angles betwee” them are equal, the resultant force will be (a) Zero (b) 10N (c) 20N (@ 10N2N ee atan ange F455 (0 ea at 3 213 13. A metal At ape i hg 8 fixed to a wall, The stressed vay fom hal by astiek. The forees Wg on the sphere are shown in the second diagram: Which of the following statements is wrone? (a) P= Wan 0 () FAP+i =! or we @T=PHH Sol. (6) T yprese Tino é w [As the metal sphere is in equilibrium under the effect of three forces therefore T+P-+W=0 From the figure Teos 0= W 20) Tsin O=P i) Ww rom equations () and id.weget P= H7ian and T* TO psa = (V0 km and tar SD (&) 10kmand ta (V5) 2 km and tan” (5) © (@ ¥52 kmand tan" WS) sultant of three ve ine following figure 4, Find the sctors. 04,08 3 BE shown int Radius of the cirel is a B [Na () Ra+¥2) “AR s.10 7 _ weF§ -Ui+jrsbaurij+ish we (ded + Loe +3 15)= 100F 1. Gi+B) is perpendicular to (4-2) ° tus G+.G-D oe #4 A-AB- = _ Because of commutative propery oft product B= B.A + AWB =000A=B Thus he ato of mages 4/8 Topicwise Sincte Connect ANswen TYPE Vector Addition oF B 2 Angle berween Ais & 7 2. (a) Ifthe angle berween all foes which are equal and lying in ‘one plane are equal, then resultant force willbe ze. 2.0 © x ¢ i) Zoxm (45° w EQ) 50 ‘Net movement along x-direction 5, (6-8) c08 45° F= 2x5 2 km ‘Net movement along y-direction L 5,=(6+4)singse J=10x4 25 Fm [Net movement from starting point IS)= fs3+ 5} = 2)? + V2)? = V52 km ‘Angle which makes withthe east direction R+VIR= RVI +), 4.(b) Ree R+ RR = 5.(d) We can observe A+ B+E=0 Priticg 7.0) FN 120" 120° 20° lp orees of equal magnitude works ona single pot ag Inv ant is zero then angle between any U0 fore, their ret given 36030 320° wos seach ofthe options (b), (c) and (4), there is one foree wick (a) In ia ‘dum of other two forces. hence resultant fee is greater than ‘cannot be zero for these. 10.(Q4=C fo = @=45 Angle between 4 and B is 180-0 = 180° — 45° 11, (a) Resultant R=P +O+P-G ‘The angle between P and 2P is zero. 12. (6) Minimum resultant = A ~ B\= 6-8) = 2kg Maximum resultant = 4=B=6+8= 14kg Only 9 kg lies berween them, 13. (a) For 17 N both the vector should be parallel i.e. angle ber them should be zero, ™ “ For 7 N both the vectors should be antiparallel ie. 282 Setwcen them shouldbe 180" 3 ‘or 13 N both the vectors should be perpendicular to other ie. angle between them should be 90°, 4.0) 64 428, The value of Clies between 4 ~ B and 4+ 8 ICl sin® + sin 7 = 1 cos! a> 1-08 B+ = 3 (cos? a+ cos? > cos =3-1=2 136. (©) Force F lie in the s-¥ plane so vector along z-axis will be perpendicular to F: cost 7 a 37.) Pos a ts According to problem [7 +7|=17; = Waal= Weal So V; and ¥ will be mutually perpendicular. 38.(ayeoso= 22-1 . ono 39. (¢) A+B AB cos 0 Inthe problem 4-2=—B ,ie4c0s 0 ie, A and B acts in the opposite direction 9= 180° @is3pd+) 41. (a) Let the components of 4 makes angles a Band ywith x, y and z axis respectively then a= B= y cos? + cos? f+ c0s? y= 1 = 30s? Ay Accra = 420) AxB=0 % sind=0 . O ‘Two vectors will be parallel to each other. 43.) Vector (P+) lies in a plane and vector (Px@) is perpendicular to this plane i.e the angle between given vectors 242)-hand B=6i -3) +26 G=AxB= (2) +2)- Hx 6-3} +28) Physics Unit vector perpendicular to both A and & je1oj—1sk__ 7-10) 18h fi? +10? +18" SVI7 wy sng BABI I 1 9n30 45.(0) sing = XBL. Io 46. (¢) If A-B=0, 2 is perpendicular to 2 0, is perpendicular to ©- cular to both Band C. 0 4 isperpen ‘Also Bx: is perpendicular to both Band @. Hence, dis parallel to BxC. 47.(0) Ax B= ABsin Of For parallel vectors 0= 0° or 180%, sin O= 0 Axb=6 48, (€) Vector perpendicular to.A and B, Ax B= AB sin 6h Unit veetor perpendicular to 4 and B je Tax iBisind 49. (a) Ax B isa vector perpendicular to plane 4+ 5 and hence perpendicular to 4+ 2 50. (dy (A+B) x (A~By= x A Ax B+ Bx A- Bx =0- 4x B+ Bx A-0=Bx A+ 5x 4-284) vsindi —mvcosj 51. (@) Pr and Pz =mvsindi +mvcos0j So change in momentum 2 Py and 5 willbe perpendicular to cach ober be perpendicular to eachother ilar to both 6 and = Bz will bea vector perpes So @ is parallel to b xé. 53. (c) pe, Fin, sind, sin 150° Hints and Solutions = 12 sin, sin 150" 2 pal Bxsinisor 99.05 nt sta) ncn 22800 Pr 2pcoxd MEO 90" = on —2psind _ P+2peoso * P*2Peos = cos0. 55.(b) ,=50) = 5, =-S0i Ab =i, soi ~50j [db] = 50V km, sw 56. (c) Bistheangle between P-+G.and B—G, Beanhaveany value from 0° to 180° depending upon magnitudes of Pand @ angle @ between them. ne owt 57.(a) Given resultant displacements km de eas may be expressed a 6 , 5e0837 5sin37" s Given displacements are (i) 2 km due east or 21 (ii) 5 km 37° south of eat its components cos37° = 5%: along x-axis, along y-axis = 5sin37°=5% (i) unknown displacement Ietitbe af +b] ajrai-ajrai +6. oivai-3j+bjaF sides ‘comparing the two and b= 3 Unknown displacement ‘58, (b) Given speed v= 50 mis in the direetion = 4 isthe unit vector in the direction 7H -2aj 24 i-24j)__ i=24)) Versa g 24) (7) -24j) m/s rox so. Initially the particle is at & =(G4-7))™ + Position of the particle after 3 sec, 7 = FeG@i-7h)+3x2101 -24)) =(45i 151) . , «im got 59, Substuting 7 =(Acosori + Asinaxj) in F=G, wehave se gt ccosanyi + att ‘ 5 aL coven + AG nO) =-dwsinanl + Acoso) =0,v=—Aasin0i + deoeos 07 = 40] A 60. (¢)In new coordinate system, Fsin3T=6N y| ON Gry, Feos 31° = 8N Lar x, y Initial coordinate system Fsin37=6N Fos 37°=8N component F, sin37?=10x2=6N 5 x F=10N component F, Numericat Value TyPE Numenicat Value Type 1.6 Since, acceleration of particle is slong X-axis. So, net forces ‘will be along X-axis. Hence, SF; Fysin37°— Fsin30° 6 eSxse FaQxseon 3.14 — 2.) dvbesi-jrk oo si-jok _si-jrk Voor 07 3.02) cos? a+ cos® + cosy =1 ‘The unit vector of A+B: Hence, a= 1 Two values of cos 7 are possible, Henee, two vectors are 1 possible, one corresponding to direction cosines =, and other corresponding to direction cosines 4.(A) cos? a+ cos" B+ cos? for 03? 60° + cos? 60"-+e% Fy =5cos37°+ Ssin3 77 = 4743) and. Fy =—4JZ costs" 4 /2sings°) =-41-4j For equilibrium ofthe particle Fish si+h no or 3hedj43i—ai-aj+h, ywi-sk Avi =S0 n=? 7.(1) Ha and b are perpendicular, a-b = 0 or 2e-3+190 Physics 8. (9) Thenet work done, Weii+hp t= thy 20143) 44h) (rg —r0) =i +3) +48)-{(5i +4]+38)- Gi +3) +40). =Q143)+48).Gi+)-)=643-4255 2.0) Iemeans @ and B are perpendicular to cach other. Sima, 1d are perpendicular to each other. So, we can say tat & and € are parallel to each othr. 4: BF =[efeos0? = 1x6= 6 units 10. (1) The volume of parallelepiped is |@-(6 >2)] =|2i-37+4b. 2h byxsi—j+28}] ij é Qi-3j+4k) jh 2 =H b -1 2 ~3)+40-{4-v-jasdeiciolf ~ 3) +4%) 1-5} —76] = [6+15-28]= 7m? 11, (4 Ifthe vectors are coplanar. Then, a-(b * c) = 0. +i)-ff00~39)~j5+9)+8-y-6) jai {a0-3yi—1s 6+yi}=0 = 110-39) +14-(6+5)=0 = 20-6y+14-6- = 28-7y=0 =0 yes 12. Q) The resultant of F and Fis in direction of 1000.8 force. So, the sum of componeat of F; and F; in perpendicular direction 0f 1000 N force will be zero, Fisin30° = F;sinas° FAlsingse ot Bo sins" 7 2 3) ab =falefoose armen fon yo" 100 Hints ond Solutions Len iT .3e, 3x17 no HOLA onal tajsaé Maxidsjx(ax})+kaxiy (ai +03) +ayk)=20 15.00) a-{b+ox(atb+o} or a{b+0xatb+oxtb+e) or a-((b+e)xa} But (6+ 6) x ais perenciculr toa 1 a-fib+eyxa] Archives 1.(¢ We know that x =~(Bx a) because the angle between these two is lays 90” But ifthe angle between Zand B is or x Then AxB=BxA=0 =04i+03) 2.(b) Given w= vewtar =3)+4]+(041+03)10 dia $38 asi sajeaiesj=1+77 So, speed is equal to magnitude of velocity VF at =n unis 3.60 Z=mexi9 ina[ ont soysno 2°] x[vgcosdi+(vosind- 20] «mace -5]é J ng cosoh 2 4.(€)For a particle in uniform circular motion, @=—> towards centre of circle 6.00)

You might also like